Infinite balls, infinite groups with finite number of balls of each value












3












$begingroup$


You have N balls of value 0, N of value 1, N of value 2, etc., N of each value up to infinity.



They are separated into a group of X balls with total value 1, another group of X with total value 2, another of X with value 3, etc., one different group of X for each total value up to infinity.



What is the smallest N for X=4 to be possible? What is the general equation for any value X?










share|cite|improve this question











$endgroup$












  • $begingroup$
    Are the balls replaced back into the box?
    $endgroup$
    – Jam
    Dec 10 '18 at 22:53












  • $begingroup$
    I don't quite understand what your question is. If at each "step", you're just removing $1$ or $4$ balls then clearly doing this up to $k$ will just require $k$ or $4k$ steps. What are you defining a "step" to be, for it to depend on $N$?
    $endgroup$
    – Jam
    Dec 10 '18 at 22:58










  • $begingroup$
    Step k is the removal of 4 balls totalling k in value. The total removed is the sum of all the steps as no balls are returned.
    $endgroup$
    – Ben Franks
    Dec 11 '18 at 8:02


















3












$begingroup$


You have N balls of value 0, N of value 1, N of value 2, etc., N of each value up to infinity.



They are separated into a group of X balls with total value 1, another group of X with total value 2, another of X with value 3, etc., one different group of X for each total value up to infinity.



What is the smallest N for X=4 to be possible? What is the general equation for any value X?










share|cite|improve this question











$endgroup$












  • $begingroup$
    Are the balls replaced back into the box?
    $endgroup$
    – Jam
    Dec 10 '18 at 22:53












  • $begingroup$
    I don't quite understand what your question is. If at each "step", you're just removing $1$ or $4$ balls then clearly doing this up to $k$ will just require $k$ or $4k$ steps. What are you defining a "step" to be, for it to depend on $N$?
    $endgroup$
    – Jam
    Dec 10 '18 at 22:58










  • $begingroup$
    Step k is the removal of 4 balls totalling k in value. The total removed is the sum of all the steps as no balls are returned.
    $endgroup$
    – Ben Franks
    Dec 11 '18 at 8:02
















3












3








3


3



$begingroup$


You have N balls of value 0, N of value 1, N of value 2, etc., N of each value up to infinity.



They are separated into a group of X balls with total value 1, another group of X with total value 2, another of X with value 3, etc., one different group of X for each total value up to infinity.



What is the smallest N for X=4 to be possible? What is the general equation for any value X?










share|cite|improve this question











$endgroup$




You have N balls of value 0, N of value 1, N of value 2, etc., N of each value up to infinity.



They are separated into a group of X balls with total value 1, another group of X with total value 2, another of X with value 3, etc., one different group of X for each total value up to infinity.



What is the smallest N for X=4 to be possible? What is the general equation for any value X?







sequences-and-series combinatorics limits






share|cite|improve this question















share|cite|improve this question













share|cite|improve this question




share|cite|improve this question








edited Dec 13 '18 at 8:29







Ben Franks

















asked Dec 10 '18 at 22:45









Ben FranksBen Franks

263110




263110












  • $begingroup$
    Are the balls replaced back into the box?
    $endgroup$
    – Jam
    Dec 10 '18 at 22:53












  • $begingroup$
    I don't quite understand what your question is. If at each "step", you're just removing $1$ or $4$ balls then clearly doing this up to $k$ will just require $k$ or $4k$ steps. What are you defining a "step" to be, for it to depend on $N$?
    $endgroup$
    – Jam
    Dec 10 '18 at 22:58










  • $begingroup$
    Step k is the removal of 4 balls totalling k in value. The total removed is the sum of all the steps as no balls are returned.
    $endgroup$
    – Ben Franks
    Dec 11 '18 at 8:02




















  • $begingroup$
    Are the balls replaced back into the box?
    $endgroup$
    – Jam
    Dec 10 '18 at 22:53












  • $begingroup$
    I don't quite understand what your question is. If at each "step", you're just removing $1$ or $4$ balls then clearly doing this up to $k$ will just require $k$ or $4k$ steps. What are you defining a "step" to be, for it to depend on $N$?
    $endgroup$
    – Jam
    Dec 10 '18 at 22:58










  • $begingroup$
    Step k is the removal of 4 balls totalling k in value. The total removed is the sum of all the steps as no balls are returned.
    $endgroup$
    – Ben Franks
    Dec 11 '18 at 8:02


















$begingroup$
Are the balls replaced back into the box?
$endgroup$
– Jam
Dec 10 '18 at 22:53






$begingroup$
Are the balls replaced back into the box?
$endgroup$
– Jam
Dec 10 '18 at 22:53














$begingroup$
I don't quite understand what your question is. If at each "step", you're just removing $1$ or $4$ balls then clearly doing this up to $k$ will just require $k$ or $4k$ steps. What are you defining a "step" to be, for it to depend on $N$?
$endgroup$
– Jam
Dec 10 '18 at 22:58




$begingroup$
I don't quite understand what your question is. If at each "step", you're just removing $1$ or $4$ balls then clearly doing this up to $k$ will just require $k$ or $4k$ steps. What are you defining a "step" to be, for it to depend on $N$?
$endgroup$
– Jam
Dec 10 '18 at 22:58












$begingroup$
Step k is the removal of 4 balls totalling k in value. The total removed is the sum of all the steps as no balls are returned.
$endgroup$
– Ben Franks
Dec 11 '18 at 8:02






$begingroup$
Step k is the removal of 4 balls totalling k in value. The total removed is the sum of all the steps as no balls are returned.
$endgroup$
– Ben Franks
Dec 11 '18 at 8:02












1 Answer
1






active

oldest

votes


















1












$begingroup$

We can show that $Nge X^2$. Indeed, for any natural $k$ consider a union $U$ of the first $kN$ groups. The sum $S$ of ball’s values of $U$ is $sum_{i=1}^{kN} i=frac 12 kN(kN+1).$
On the other hand, for each $j$, $U$ contains at most $N$ balls with value $j$. Since $|U|=kXcdot N$, we have $Sge Nsum_{j=0}^{kX-1} j=frac 12 N(kX-1)kX.$ Thus $$frac 12 kN(kN+1)ge frac 12 N(kX-1)kX$$ or $kN+1ge kX^2-X$. Since this inequality hold for all $k$, we have $Nge X^2$.






share|cite|improve this answer









$endgroup$













  • $begingroup$
    do you have an upper bound to go with this lower bound?
    $endgroup$
    – Ben Franks
    Dec 18 '18 at 15:46










  • $begingroup$
    @BenFranks Unortunately, no. I think it should be obtained by a different way, maybe by come combinatorial constructions.
    $endgroup$
    – Alex Ravsky
    Dec 18 '18 at 15:48











Your Answer





StackExchange.ifUsing("editor", function () {
return StackExchange.using("mathjaxEditing", function () {
StackExchange.MarkdownEditor.creationCallbacks.add(function (editor, postfix) {
StackExchange.mathjaxEditing.prepareWmdForMathJax(editor, postfix, [["$", "$"], ["\\(","\\)"]]);
});
});
}, "mathjax-editing");

StackExchange.ready(function() {
var channelOptions = {
tags: "".split(" "),
id: "69"
};
initTagRenderer("".split(" "), "".split(" "), channelOptions);

StackExchange.using("externalEditor", function() {
// Have to fire editor after snippets, if snippets enabled
if (StackExchange.settings.snippets.snippetsEnabled) {
StackExchange.using("snippets", function() {
createEditor();
});
}
else {
createEditor();
}
});

function createEditor() {
StackExchange.prepareEditor({
heartbeatType: 'answer',
autoActivateHeartbeat: false,
convertImagesToLinks: true,
noModals: true,
showLowRepImageUploadWarning: true,
reputationToPostImages: 10,
bindNavPrevention: true,
postfix: "",
imageUploader: {
brandingHtml: "Powered by u003ca class="icon-imgur-white" href="https://imgur.com/"u003eu003c/au003e",
contentPolicyHtml: "User contributions licensed under u003ca href="https://creativecommons.org/licenses/by-sa/3.0/"u003ecc by-sa 3.0 with attribution requiredu003c/au003e u003ca href="https://stackoverflow.com/legal/content-policy"u003e(content policy)u003c/au003e",
allowUrls: true
},
noCode: true, onDemand: true,
discardSelector: ".discard-answer"
,immediatelyShowMarkdownHelp:true
});


}
});














draft saved

draft discarded


















StackExchange.ready(
function () {
StackExchange.openid.initPostLogin('.new-post-login', 'https%3a%2f%2fmath.stackexchange.com%2fquestions%2f3034601%2finfinite-balls-infinite-groups-with-finite-number-of-balls-of-each-value%23new-answer', 'question_page');
}
);

Post as a guest















Required, but never shown

























1 Answer
1






active

oldest

votes








1 Answer
1






active

oldest

votes









active

oldest

votes






active

oldest

votes









1












$begingroup$

We can show that $Nge X^2$. Indeed, for any natural $k$ consider a union $U$ of the first $kN$ groups. The sum $S$ of ball’s values of $U$ is $sum_{i=1}^{kN} i=frac 12 kN(kN+1).$
On the other hand, for each $j$, $U$ contains at most $N$ balls with value $j$. Since $|U|=kXcdot N$, we have $Sge Nsum_{j=0}^{kX-1} j=frac 12 N(kX-1)kX.$ Thus $$frac 12 kN(kN+1)ge frac 12 N(kX-1)kX$$ or $kN+1ge kX^2-X$. Since this inequality hold for all $k$, we have $Nge X^2$.






share|cite|improve this answer









$endgroup$













  • $begingroup$
    do you have an upper bound to go with this lower bound?
    $endgroup$
    – Ben Franks
    Dec 18 '18 at 15:46










  • $begingroup$
    @BenFranks Unortunately, no. I think it should be obtained by a different way, maybe by come combinatorial constructions.
    $endgroup$
    – Alex Ravsky
    Dec 18 '18 at 15:48
















1












$begingroup$

We can show that $Nge X^2$. Indeed, for any natural $k$ consider a union $U$ of the first $kN$ groups. The sum $S$ of ball’s values of $U$ is $sum_{i=1}^{kN} i=frac 12 kN(kN+1).$
On the other hand, for each $j$, $U$ contains at most $N$ balls with value $j$. Since $|U|=kXcdot N$, we have $Sge Nsum_{j=0}^{kX-1} j=frac 12 N(kX-1)kX.$ Thus $$frac 12 kN(kN+1)ge frac 12 N(kX-1)kX$$ or $kN+1ge kX^2-X$. Since this inequality hold for all $k$, we have $Nge X^2$.






share|cite|improve this answer









$endgroup$













  • $begingroup$
    do you have an upper bound to go with this lower bound?
    $endgroup$
    – Ben Franks
    Dec 18 '18 at 15:46










  • $begingroup$
    @BenFranks Unortunately, no. I think it should be obtained by a different way, maybe by come combinatorial constructions.
    $endgroup$
    – Alex Ravsky
    Dec 18 '18 at 15:48














1












1








1





$begingroup$

We can show that $Nge X^2$. Indeed, for any natural $k$ consider a union $U$ of the first $kN$ groups. The sum $S$ of ball’s values of $U$ is $sum_{i=1}^{kN} i=frac 12 kN(kN+1).$
On the other hand, for each $j$, $U$ contains at most $N$ balls with value $j$. Since $|U|=kXcdot N$, we have $Sge Nsum_{j=0}^{kX-1} j=frac 12 N(kX-1)kX.$ Thus $$frac 12 kN(kN+1)ge frac 12 N(kX-1)kX$$ or $kN+1ge kX^2-X$. Since this inequality hold for all $k$, we have $Nge X^2$.






share|cite|improve this answer









$endgroup$



We can show that $Nge X^2$. Indeed, for any natural $k$ consider a union $U$ of the first $kN$ groups. The sum $S$ of ball’s values of $U$ is $sum_{i=1}^{kN} i=frac 12 kN(kN+1).$
On the other hand, for each $j$, $U$ contains at most $N$ balls with value $j$. Since $|U|=kXcdot N$, we have $Sge Nsum_{j=0}^{kX-1} j=frac 12 N(kX-1)kX.$ Thus $$frac 12 kN(kN+1)ge frac 12 N(kX-1)kX$$ or $kN+1ge kX^2-X$. Since this inequality hold for all $k$, we have $Nge X^2$.







share|cite|improve this answer












share|cite|improve this answer



share|cite|improve this answer










answered Dec 15 '18 at 11:53









Alex RavskyAlex Ravsky

40.5k32282




40.5k32282












  • $begingroup$
    do you have an upper bound to go with this lower bound?
    $endgroup$
    – Ben Franks
    Dec 18 '18 at 15:46










  • $begingroup$
    @BenFranks Unortunately, no. I think it should be obtained by a different way, maybe by come combinatorial constructions.
    $endgroup$
    – Alex Ravsky
    Dec 18 '18 at 15:48


















  • $begingroup$
    do you have an upper bound to go with this lower bound?
    $endgroup$
    – Ben Franks
    Dec 18 '18 at 15:46










  • $begingroup$
    @BenFranks Unortunately, no. I think it should be obtained by a different way, maybe by come combinatorial constructions.
    $endgroup$
    – Alex Ravsky
    Dec 18 '18 at 15:48
















$begingroup$
do you have an upper bound to go with this lower bound?
$endgroup$
– Ben Franks
Dec 18 '18 at 15:46




$begingroup$
do you have an upper bound to go with this lower bound?
$endgroup$
– Ben Franks
Dec 18 '18 at 15:46












$begingroup$
@BenFranks Unortunately, no. I think it should be obtained by a different way, maybe by come combinatorial constructions.
$endgroup$
– Alex Ravsky
Dec 18 '18 at 15:48




$begingroup$
@BenFranks Unortunately, no. I think it should be obtained by a different way, maybe by come combinatorial constructions.
$endgroup$
– Alex Ravsky
Dec 18 '18 at 15:48


















draft saved

draft discarded




















































Thanks for contributing an answer to Mathematics Stack Exchange!


  • Please be sure to answer the question. Provide details and share your research!

But avoid



  • Asking for help, clarification, or responding to other answers.

  • Making statements based on opinion; back them up with references or personal experience.


Use MathJax to format equations. MathJax reference.


To learn more, see our tips on writing great answers.




draft saved


draft discarded














StackExchange.ready(
function () {
StackExchange.openid.initPostLogin('.new-post-login', 'https%3a%2f%2fmath.stackexchange.com%2fquestions%2f3034601%2finfinite-balls-infinite-groups-with-finite-number-of-balls-of-each-value%23new-answer', 'question_page');
}
);

Post as a guest















Required, but never shown





















































Required, but never shown














Required, but never shown












Required, but never shown







Required, but never shown

































Required, but never shown














Required, but never shown












Required, but never shown







Required, but never shown







Popular posts from this blog

Quarter-circle Tiles

build a pushdown automaton that recognizes the reverse language of a given pushdown automaton?

Mont Emei